RSA Encryption Problem: Finding d Using Euclidean Algorithm

  • Thread starter Thread starter muso07
  • Start date Start date
  • Tags Tags
    Encryption
Click For Summary
To find d in RSA encryption, the multiplicative inverse of e (3) modulo (p-1)(q-1) (17680) is required. The extended Euclidean algorithm is used to compute this inverse, leading to the equation 1 = 17680 - 5893(3). Rearranging gives d = -5893, which is equivalent to 11787 when taken modulo 17680. This confirms that d = 11787 is the correct value for the decryption exponent in this RSA setup. Understanding the steps of the Euclidean algorithm is crucial for solving similar problems.
muso07
Messages
53
Reaction score
0

Homework Statement


This is a problem my lecturer did in class but I'm a bit confused.

We have 2 large primes p=137 and q=131, choose e = 3
n = pq = 137.131 = 17947
(p-1)(q-1) = 136.130 = 17680

Compute d = e-1 mod (p-1)(q-1) = 3-1 mod 17680 = 11787.

This is where I'm confused. How did he get d=11787?

Homework Equations

The Attempt at a Solution


I think I'm supposed to use the Euclidean algorithm to find d, but he didn't show how.
 
Last edited:
Physics news on Phys.org
for d to be the multiplicative inverse of 3 mod 17680 you need 3*d = 1 mod 17680; there will be a unique positive d < 17680 for which this is true, and this d can be found using the extended euclidean algorithm. the following page will be helpful:
http://www-math.cudenver.edu/~wcherowi/courses/m5410/exeucalg.html

for your problem you will start
17680 = 5893(3) + 1
and the euclidean algorithm terminates quickly because
3 = 3(1) + 0
rearranging the line above you get
1 = 17680 - 5893(3)
so you know that d = -5893, which is 11787 mod 17680.
 
Last edited by a moderator:
Lindsay Childs in "A Concrete Introduction to Higher Algebra" deals with RSA details if you need more information.
 
Question: A clock's minute hand has length 4 and its hour hand has length 3. What is the distance between the tips at the moment when it is increasing most rapidly?(Putnam Exam Question) Answer: Making assumption that both the hands moves at constant angular velocities, the answer is ## \sqrt{7} .## But don't you think this assumption is somewhat doubtful and wrong?

Similar threads

  • · Replies 2 ·
Replies
2
Views
2K
  • · Replies 7 ·
Replies
7
Views
2K
  • · Replies 1 ·
Replies
1
Views
958
  • · Replies 1 ·
Replies
1
Views
1K
  • · Replies 3 ·
Replies
3
Views
1K
  • · Replies 2 ·
Replies
2
Views
1K
Replies
6
Views
2K
Replies
8
Views
3K
  • · Replies 5 ·
Replies
5
Views
1K
  • · Replies 16 ·
Replies
16
Views
3K